LSAT--Columnist

This topic has expert replies
User avatar
Junior | Next Rank: 30 Posts
Posts: 23
Joined: Sun Sep 12, 2010 7:45 am
Thanked: 4 times

by mzkfrk » Wed Aug 03, 2011 10:31 pm
DanaJ wrote:Received a PM.

The reasoning goes as follows:
- there have been notable improvements in the quality of the environment
- this supports the idea that you still need to have some economical activity to support the costs associated with environmental improvements

The conclusion here is that wealth generated by the use of resources can be used for improving the environment.

A is spot on. If the wealth of a nation is generated by the use of its resources, then we actually do need to use these resources or else we can't pay for the environmental improvements.

B may be true, but there's no mentioning of the costs. What if technology is not that expensive (I know it sounds silly, but you need to take into account only what's given and avoid making assumptions)? Besides, social programs do not necessarily mean environmental programs.

C - could be true, but does not affect the argument of wealth and environmental improvements.

D - could be true, but does not reference the conclusion, i.e. no mentioning of wealth.

E - this is actually the "reverse argument" option. This option states that concerns for the environment lead to greater wealth. The conclusion, however, has it the other way around. So this option would weaken the argument because it says that the reasoning is flawed (i.e. backwards).
o
I have a doubt. Option A states that nation sustains it's wealth through the resources but the main statement never talks about the nation, it says environmentalists won't be able to generate wealth to implement the policies. How do you relate the two?

That's why I chose D.
"Kites rise highest against the wind, not with it"
- Sir Winston Churchill

Senior | Next Rank: 100 Posts
Posts: 72
Joined: Wed Oct 27, 2010 2:06 am
Thanked: 2 times
Followed by:1 members

by saurabh2525_gupta » Sun Aug 21, 2011 7:42 am
mzkfrk wrote:
DanaJ wrote:Received a PM.

The reasoning goes as follows:
- there have been notable improvements in the quality of the environment
- this supports the idea that you still need to have some economical activity to support the costs associated with environmental improvements

The conclusion here is that wealth generated by the use of resources can be used for improving the environment.

A is spot on. If the wealth of a nation is generated by the use of its resources, then we actually do need to use these resources or else we can't pay for the environmental improvements.

B may be true, but there's no mentioning of the costs. What if technology is not that expensive (I know it sounds silly, but you need to take into account only what's given and avoid making assumptions)? Besides, social programs do not necessarily mean environmental programs.

C - could be true, but does not affect the argument of wealth and environmental improvements.

D - could be true, but does not reference the conclusion, i.e. no mentioning of wealth.

E - this is actually the "reverse argument" option. This option states that concerns for the environment lead to greater wealth. The conclusion, however, has it the other way around. So this option would weaken the argument because it says that the reasoning is flawed (i.e. backwards).
o
I have a doubt. Option A states that nation sustains it's wealth through the resources but the main statement never talks about the nation, it says environmentalists won't be able to generate wealth to implement the policies. How do you relate the two?

That's why I chose D.
The way I look at the conclusion is:-

The change may be due to efforts of the environmentalists but people claim that excess restrictive use will lessen the ability to generate wealth(who's ability - not explicitly specified). Since the author is not taking anyone' name here we can take it as the nation itself.

Hope this helps!!!

Best Regards,
JOHN

Legendary Member
Posts: 608
Joined: Sun Jun 19, 2011 11:16 am
Thanked: 37 times
Followed by:8 members

by saketk » Mon Aug 22, 2011 6:08 pm
The answer should be Option A. I was confused between D and A, ultimately chose A because it supports the conclusion of the columnist better

User avatar
Senior | Next Rank: 100 Posts
Posts: 53
Joined: Sun Apr 10, 2011 12:15 pm
Thanked: 1 times

by Deependra1 » Wed Aug 31, 2011 11:37 am
ANSWER:D

User avatar
Legendary Member
Posts: 540
Joined: Sat Dec 20, 2008 7:24 pm
Thanked: 37 times
Followed by:6 members

by navami » Sat Sep 03, 2011 11:07 pm
IMO A
This time no looking back!!!
Navami

Master | Next Rank: 500 Posts
Posts: 101
Joined: Thu Aug 25, 2011 9:39 pm

by prashant misra » Sun Sep 04, 2011 11:20 pm
thank you dana for your proper explanation.i chose the wrong option once again but got my doubt clear through your explanation

Junior | Next Rank: 30 Posts
Posts: 24
Joined: Sun Sep 18, 2011 9:48 am

by asgupta2k » Wed Sep 28, 2011 1:57 pm
D is the correct answer because it tell the exact reason why the environmentalist policy should not implemented and how it can lead to less improvement in environment.

Newbie | Next Rank: 10 Posts
Posts: 1
Joined: Wed Oct 05, 2011 8:55 pm

by kapil.rapelli » Sun Oct 16, 2011 6:22 pm
Dana,

Option A introduces a new term "within their boundaries" which is not explicitly mentioned in the argument. How do we handle such options??

Senior | Next Rank: 100 Posts
Posts: 90
Joined: Thu Nov 05, 2009 9:14 am
Thanked: 5 times
Followed by:3 members

by immaculatesahai » Thu Nov 10, 2011 2:01 am
Wow, last line of the CR stimulus had me reading it twice. But the moment you understand it, A is the obvious answer. Good one.

Senior | Next Rank: 100 Posts
Posts: 85
Joined: Tue Sep 02, 2008 12:13 am
Thanked: 1 times
GMAT Score:650

by vzzai » Thu Nov 10, 2011 4:27 am
Tough one for me, I took 4+mins but arrived at A.
Is it only who finds this q'n little convoluted in wording?
Thank you,
Vj

Junior | Next Rank: 30 Posts
Posts: 21
Joined: Mon Jul 20, 2009 11:57 pm

by ReyWilli » Fri Dec 09, 2011 12:31 pm
I think B.
Conclusion "it lends credibility to people who reject ecological doom...and argue that policies that restrict natural resources diminish wealth..."
It took me really long to understand what this was saying. I tried breaking it into pieces to help.

I think we're looking for answer that shows not restricting resources is good.

A) Not showing increase of used resource
B) keyword: "more advanced technology...greater ability devote to resources". Shows increase in technology helped a social program policy
C) Doesn't demonstrate increase in resource helps
D) not concerned about compromise
E) This sounds like weakens. Argument saying not to restrict.
GMATJunky

User avatar
Master | Next Rank: 500 Posts
Posts: 296
Joined: Sun May 29, 2011 5:10 am
Location: Vietnam
Thanked: 10 times
Followed by:5 members

by tuanquang269 » Mon Dec 12, 2011 2:57 am
it nonetheless
lends credibility to the claims of people who
reject predictions of imminent ecological doom
and argue that environmental policies that
excessively restrict the use of natural resources
may diminish the wealth necessary to adopt and
sustain the policies that brought about these
improvements.

Choice B is reverse " "more advanced technology...greater ability devote to resource". We need utilizing natural resource to improve.

Choice A is better.

Master | Next Rank: 500 Posts
Posts: 160
Joined: Tue Jul 07, 2009 1:09 pm
Thanked: 1 times
Followed by:1 members

by Sharma_Gaurav » Sun Jan 08, 2012 11:54 am
selected option A , but took more than 2 minutes.
I want to improve the speed:(

Senior | Next Rank: 100 Posts
Posts: 63
Joined: Sat Dec 24, 2011 9:47 am
Location: India

by him1985 » Thu Jan 26, 2012 3:40 am
I will go with D
As paragraph revolves around "Heavily forested now/ Due to policies"... Further paragraph is supporting to its 1st two lines.
And D is strongly supporting the centre theme of paragraph.

:)
Himanshu Chauhan

User avatar
Junior | Next Rank: 30 Posts
Posts: 12
Joined: Mon Jan 23, 2012 10:31 am

by bostonblue » Fri Feb 03, 2012 1:21 pm
I'll be honest I wasn't immediately sure what the columnist was arguing, but I settled on A.